LSAT and Law School Admissions Forum

Get expert LSAT preparation and law school admissions advice from PowerScore Test Preparation.

 Administrator
PowerScore Staff
  • PowerScore Staff
  • Posts: 8917
  • Joined: Feb 02, 2011
|
#34810
Complete Question Explanation

Strengthen. The correct answer choice is (E)

This argument has elements of Malthusian thinking within it, namely that at some point the population will exceed our ability to produce food.

The expert opens this stimulus using the “some people say...” technique, first described in our discussion about Question #5. Here, the expert presents the view of “some people,” who argue that there is no need for concern about food shortages, because so far food production has increased faster than the population has. As expected, the expert disagrees with these people, concluding that “widespread food shortages are inevitable.” In support of this view, the expert says that the planet’s resources can support food production at only a few times the current amount and, after that limit is reached, there can be no more increases in production.

The expert’s argument is flawed because it assumes that the population will increase to a point at which the maximum food production level will be insufficient to support the population. The question stem identifies this as a Strengthen question, the third of the section thus far. Our prephrase is that the correct answer choice will strengthen the expert’s conclusion by providing evidence supporting the assumption that the population would grow beyond the planet’s maximum food production capacity.

Answer choice (A): This answer choice addresses the issue of sustainability, not capacity. However, the argument is centered on the planet’s maximum food production capacity.

Answer choice (B): In the stimulus, the expert referred to the “planet’s resources,” which would include food resources from the oceans. This statement adds nothing new to the argument.

Answer choice (C): This answer choice weakens the expert’s conclusion by casting doubt on whether the planet’s population will continue to increase.

Answer choice (D): The historical occurrence of regional food shortages is irrelevant to the conclusion, which dealt with the sufficiency of maximal planetary food production.

Answer choice (E): This is the correct answer choice, because it restates our prephrase, that the planet’s population will grow beyond the maximum level of food production.
 kfactor901@gmail.com
  • Posts: 11
  • Joined: Aug 27, 2016
|
#28230
Hello, I was wondering if someone can explain exactly how E strengthens the argument? The conclusion is that the people's claims are false and that widespread food shortages are inevitable. I chose B because it seemed like that once source of food will be completely used up which can lead to some shortages. Thank you!

-K
 David Boyle
PowerScore Staff
  • PowerScore Staff
  • Posts: 836
  • Joined: Jun 07, 2013
|
#28242
kfactor901@gmail.com wrote:Hello, I was wondering if someone can explain exactly how E strengthens the argument? The conclusion is that the people's claims are false and that widespread food shortages are inevitable. I chose B because it seemed like that once source of food will be completely used up which can lead to some shortages. Thank you!

-K

Hello -K,

Answer B, "Food resources from the world’s oceans will eventually be fully utilized", is already somewhat implied when the stimulus says, "the planet’s resources allow for food to be produced at only a few times the current amount, beyond which no increase in production will be possible". But that limit on production might not be a problem if there weren't too many people. Answer E shows that there may indeed be too many people for the food supply, "Population will continue to grow at least briefly when food production has reached its maximum level."

Hope this helps,
David
 c-erv
  • Posts: 15
  • Joined: Oct 14, 2016
|
#29991
Hello, I also selected B as the right answer and I am having trouble seeing why E is the right answer. When I read E I view it has having no effect on the conclusion.
 David Boyle
PowerScore Staff
  • PowerScore Staff
  • Posts: 836
  • Joined: Jun 07, 2013
|
#30056
c-erv wrote:Hello, I also selected B as the right answer and I am having trouble seeing why E is the right answer. When I read E I view it has having no effect on the conclusion.

Hello c-erv,

"Thus, widespread food shortages are inevitable" is the conclusion, and E strengthens that because it has the population growing, rather than being static or shrinking.

Hope this helps,
David
 hassan66
  • Posts: 51
  • Joined: Jul 19, 2018
|
#60072
Hi, I also chose B, however, the way I see it now, even if all the food resources from the oceans will be used up, there are still other ways people can get food. So it does not necessarily indicate that food shortages are inevitable. However, with E, if the population will continue to grow even when the food production is maxed, then it will be inevitable that there will be a food shortage.
User avatar
 Dave Killoran
PowerScore Staff
  • PowerScore Staff
  • Posts: 5852
  • Joined: Mar 25, 2011
|
#60219
hassan66 wrote:Hi, I also chose B, however, the way I see it now, even if all the food resources from the oceans will be used up, there are still other ways people can get food. So it does not necessarily indicate that food shortages are inevitable. However, with E, if the population will continue to grow even when the food production is maxed, then it will be inevitable that there will be a food shortage.
Hi Hassan!

Yes, I'd agree with what you say about (E), but I'd add about (B) that to a certain extent the Expert already addressed the planet's resources in saying that "the planet’s resources allow for food to be produced at only a few times the current amount." Thus, noting that the ocean resources will eventually be fully utilized doesn't really tell us anything new. The Expert already believes (B) and basically says that once that happens, we're in trouble. It similar to the following:

  • Expert: When I go to the store next week, I can buy a candy bar.

    Answer (B): Eventually you will go to the store.
That answer really doesn't add anything to what we are talking about :-D

Please let me know if that helps. Thanks!

Get the most out of your LSAT Prep Plus subscription.

Analyze and track your performance with our Testing and Analytics Package.